Pre-Exam 2021 - claims analysis part: our answers

As in the legal part, when you check your answers with ours, note that the order of the statements within a question may be different!

Please feel invited to comment!

Note: the legal part is discussed in our other blog post: here
First impressions and general comments can be posted here.

[Update 4 March 2021 7:15:] We have compiled a version of the paper, in English; it is available hereIn our version, the sequence of the questions and statements corresponds to the order as was presented to some candidates; other candidates obtained the questions (in the legal parts) and the statements within a question (in the legal parts as well as in the claims analysis parts) in different order. The sequence/numbering of the questions in our version of the paper corresponds to that used in this blog post.

Our claims analysis answers are given below.

[Update 2 April 2021 8:45:] 
The Examiner's Report was published yesterday on the EQE website (Compendium); all our answers to Q.11-Q.20 correspond to the answers in the Examiner's Report, possibly except 12.3. No statements were neutralized in the claims analysis part, except possibly 12.3 (see below).

[Update 7 April 2021, 8:30:]
The Examiner's Report on the Compendium pages has been updated: the answer given in the updated version of the Examiner's Report to 12.3 (12.4 in the Examiner's Report) is FALSE.

[Update 8 Feb 2022:]
Today, decision D 2/21 was published. The Disciplinary Board of Appeal reasoned:
"17. For the above reasons, the answer "False" cannot be considered to be the only correct answer that can be given to statement 12.4 [corresponding to 12.3 in the version on this blog] when taking an informed and objective view or interpretation of the wording of the facts underlying the pre-examination question 12 and the statement 12.4. As a consequence, the question of whether or not statement 12.4 is correct cannot be answered with either "True" or "False" as required by a "multiple-choice" question in the pre-examination."



Part 3 - A glass composition for use as a photochromic lens

[Update 8 March 2021: added the claim sets]


QUESTION 11 -

Set of claims I, used in several questions:


I-1. A glass composition for use as a photochromic lens comprising:

silicon dioxide;

component A;

component M; and

component X.

I-2. The composition according to claim I-1 wherein component A is present in the composition in a content of about 10% to about 30% by weight.

I-3. The composition according to claim I-1 or claim I-2 wherein component A is selected from the group consisting of sodium oxide, sodium carbonate and boron trioxide.

I-4. The composition according to claim I-1 wherein component M is present in the composition in a content of 5% to 10% by weight.

I-5. The composition according to claim I-4 wherein component M is present in the composition in a content of 15% by weight.

I-6. The composition according to any one of claims I-1 to I-5, wherein component X is selected from the group consisting of compound alpha, compound beta and compound gamma.

I-7. The process according to any one of claims I-1 to I-6, wherein component M is selected from the group consisting of: lime, zinc oxide and alumina.

I-8. A process for manufacturing a photochromic lens from the glass composition according to any one of claims I-1 to I-7, which comprises the following steps:

(i) mixing components A and M together in a first vessel to obtain a first mixture;

(ii) heating silicon dioxide in a second vessel to an high temperature;

(iii) adding the first mixture from step (i) to the second vessel from step (ii) to form a second mixture;

(iv) adding component X to the second mixture;

(v) forming a photochromic lens; and afterwards

(vi) allowing the photochromic lens to cool to a low temperature.

I-9. A photochromic lens obtainable by the process of claim I-8.


11.1 Claim I-1 is limited to a glass composition which is only for use as a photochromic lens

    False: “for use as a photochromic lens” is to be interpreted as ‘suitable for use as’ but which allows other uses.

11.2 The protection of claim I-8 extends to the photochromic lens directly obtained by the process

    True: According to Art 64(2) EPC/Art. 28(1)(b) TRIPS, protection of a process claim extends to the directly obtained product by that process. A manufacturing claim is a process claim.

11.3 A photochromic lens which is identical to the photochromic lens of claim I-9, but which is not obtained by a process of claim I-8, falls under the scope of claim I-9

    True: Product-by-process claim. Here, the method steps are only used to define the properties of the product, but are otherwise not limiting.

11.4 Claim I-4 covers a glass composition for use as a photochromic lens, comprising silicon dioxide, sodium oxide in a content of 10% to 30% by weight of the composition, component M in a content of 5% to 10% by weight of the composition, and component X

    True: The claim does not specify features/limitations which are not present in the given glass composition (sodium oxide is an example (type of) of A)

 

12.1 Claim I-5 is clear

    False: Claim I-5 is dependent on claim I-4 which already limits M to 5-10% by weight. The additional feature of claim I-5 is thus contradictory with claim I-4.

12.2 Claim I-7 is unclear due to its dependency on claims I-1 to I-6

    True: Claims I-1 to I-6 are composition (product claims) and do not relate to a process; a claim stating “the process according to any one of claims I-1 to I-6” is thus unclear.

12.3 Claim I-8 is unclear due to the use of the term low.

[Update 2 April 2021 8:45:] No answer indicated in Examiner's Report. The reasoning in Examiner's Report suggests FALSE, but the absence of an answer suggests that it may be neutralized or is still subject to review. The reasoning given in the Examiner's Report of 1 April 2021 (wherein this is statement 12.4) provides:

"The term “low” in feature (vi) in claim I-8 is clearly distinguished from the term “high” in feature (ii) of claim I-8. Furthermore, the description of the application defines the term “high” in paragraph [009]. Accordingly, in the context of the whole disclosure of the application, the term “low” is considered to be clear.

In any case, if a relative term is not the only feature to distinguish the subject- matter of a claim from the prior art, the use of the relative term may not be objected to under Article 84 EPC. The word “low” is not the only feature to distinguish the claim from the prior art, as none of the prior art discloses the claimed process. Although broad, the term is not necessarily unclear [GL 2019 F-IV 4.6.1]."

The Examiner's report argumentation is discussed in the comments to our "Results" blog (here). Please feel invited to contribute to the discussion there.

[Update 7 April 2021, 8:30:]
The Examiner's Report on the Compendium pages has been updated: the answer given in the updated version of the Examiner's Report to 12.3 (12.4 in the Examiner's Report) is FALSE.

Our original answer and notes:

    True: ‘low’ is an undefined relative term; in addition, the description does not indicate that ‘low temperature’ has a generally accepted meaning in the art.

    Note: the claim does not only use the term low" but also the term "high" so that it may argued that "low" is not a relative term as such, but relative to the other "high". But even when considering that, the claim is not clear to to the term "low": Refer to GL F-IV 4.6.2 "Clarity objections", which provides: 

"Relative or similar terms such as "thin", "wide" or "strong" constitute a potentially unclear element due to the fact that their meaning may change depending on the context. For these terms to be allowed, their meaning must be clear in the context of the whole disclosure of the application or patent." 

While the application does provide preferred ranges for a "high" temperature, there is not a single range or value mentioned of what would be a "low temperature". So, one cannot conclude whether "low" means less than room temperature, even lower, or just a bit less than "high" while still several hundreds degC (quite high), not even when considering the whole disclosure, such that the statement "Claim I-8 is unclear due to the use of the term low" (but not so much due to the term high) is TRUE. [Updated 17 March 2021]

12.4 Claim I-1 contains all essential features of the glass composition of the invention

    False: According to [009] of the client’s application, it is essential that the content of component M is between about 5% to about 10% by weight. This thus represents an essential feature.

 

13.1 D1 discloses a component X being selected from the group consisting of compound alpha, compound beta and compound gamma

    False: D1 does not disclose any species/examples of X. Disclosure of X as a genus does not imply disclosure of its species.

13.2 The subject-matter of claim I-1 lacks novelty over D1

    False: D1 shows the combination of A, M and X only as a selection from three lists (‘multiple list selection’). The specific combination is thereby not disclosed by D1.

13.3 The subject-matter of claim I-2 lacks novelty over Example 2 of D2

    True: D2 second example’s disclosure of 30% by weight of A anticipates the claimed end-point of the range of about 10% to about 30% of A.

13.4 The subject-matter of claim I-4 is novel over Example 3 of D2

    False: D2 third example’s disclosure of 7% by weight of M falls within the claimed range of 5-10%.

 

QUESTION 14

During examination the applicant files a new set of claims II to replace the originally filed claim set I:

 

II-1. A glass composition for use as a photochromic lens comprising:

silicon dioxide;

component A; and

component M.

II-2. The composition according to claim II-1 further comprising component X.

II-3. The composition according to claim II-2 wherein component M is not zinc oxide.

II-4. The composition according to claim II-2 wherein component A is present in the composition in a content of 10% to 30% by weight.

II-5. The composition according to claim II-2 wherein component X is selected from the group consisting of compound alpha and compound beta.


14.1 Claim II-3 is allowable under Article 123(2) EPC

    True: True, claim II-3 corresponds to claim I-1 but with an allowable disclaimer (‘not zinc oxide’) according to G2/10.

14.2 Claim II-1 is allowable under Article 123(2) EPC

    False: The claim omits component X but which according to [011] provides the photochromic characteristics to which the invention pertains; there is no mention X can be omitted in a glass composition for a photochromic lens; its omission is thereby not directly and unambiguously derivable from the application.

14.3 Claim II-4 is allowable under Article 123(2) EPC

    True: The combination of claims II-4, II-2 and II-1 corresponds to originally filed claim I-2.

14.4 Claim II-5 is allowable under Article 123(2) EPC

    True: The combination of claims II-5, II-2 and II-1 is based on originally filed claim I-6 when directly dependent on claim I-1 and claiming two of the three options (alpha, beta, gamma)


QUESTION 15

A further amended set of claims is filed during examination:

 

CLAIM SET III

III-1. A glass composition for use as a photochromic lens comprising:

silicon dioxide;

component A which is present in a content of 16% by weight;

component M which is present in a content of 5% to 10% by weight; and

compound alpha which is present in a content of 10% by weight.

III-2. The composition according to claim III-1 wherein component A is selected from the group consisting of sodium oxide, sodium carbonate and boron trioxide.

III-3. The composition according to claim III-1 or claim III-2 wherein component M is selected from the group consisting of lime and alumina.

  

15.1 A valid argument for why the subject-matter of claim III-1 has an inventive step over D2 is because using compound alpha as component X results in a photochromic lens, the colour of which is surprisingly more resistant to temperature changes than when compound beta or compound gamma is used

    True: D2 only discloses X in form of beta and gamma; according to [012], these do not have the advantageous effect of providing resistance to color changes due to temperature changes.

15.2 A combination of D2 and D3 could be used to validly attack claim III-1 for lack of inventive step

    False: D3 is Art. 54(3) and thus not part of the state of the art for inventive step.

15.3 D2 can be considered to be the closest prior art for claim III-3 because it relates to the same technical field, i.e. the field of photochromic lenses

    True(*): D2 is indeed from the same field; D1 is from a more remote field being glass compositions for bottles.

    (*) False can also be argued: see comment below

     [Update 2 April 2021 8:45:]
     The statement was not neutralized in the Examiner's Report. Only True was considered correct.

15.4 The objective technical problem to be solved by the subject-matter of claim III-1 with respect to D2 is the provision of a photochromic lens, the colour change of which is resistant to temperature changes

    True(*): This is indeed the technical effect of using alpha as X.

     (*) False can also be argued: see comment below.

     [Update 2 April 2021 8:45:]
     The statement was not neutralized in the Examiner's Report. Only True was considered correct.
     The Examiner's report argumentation is discussed in the comments to our "Results" blog (here).
     Please feel invited to contribute to the discussion there.

(*) Your answer to 15.3 and 15.4  may depend on whether you interpreted claim III-1 as in essence defining a photochromic lens, or rather as defining a glass composition which is a precursor to a photochromic lens (e.g., by still having to be manufactured from a glass composition, as defined by the preamble and step (v) of claim I-8). We more or less expect the intended answers to be ‘true’ . 
However, there are certainly good arguments to be made for ‘false’; in particular doe [015] clearly define that a composition is obtained from mixing components and that this mixture (the composition) is the formed into a photochromic lens: the lens can thus not be consider to be a composition, but is is made from a composition. It may thus be argued that the composition is rather a composition for use in making a photochromic lens or a composition for a photochromic lens, than as a composition shaped to be used as a photochromic lens. It would be like "a plastic used as a cup": such phrasing is unclear as to whether it defines a plastic usable to make a cup from, or as a cup made out of plastic.
Also see various comments posted to this blog w.r.t. 15.3 and 15.4. 
[Expanded 5 March 2021]


Part 4 - Protection device comprising a receiving part for receiving a tablet computer and a cover part

 

The following set of claims IV was filed together with the second European patent application. This set of claims IV will be used in several questions:

IV.1 Protection device comprising a receiving part for receiving a tablet computer and a cover part.

IV.2 Protection device according to claim IV.1, wherein the protection device comprises a hinge between the receiving part and the cover part.

IV.3 Protection device according to claim IV.1 or IV.2, wherein the receiving part comprises a flat base, preferably as shown in Fig.1.

IV.4 Protection device according to any preceding claim, wherein the receiving part comprises an elastically deformable means for securing the tablet computer within the receiving part.

IV.5 Protection device according to claim IV.4, wherein the elastically deformable means comprises four elastically deformable corners.

IV.6 Protection device according to any preceding claim, wherein the cover part comprises a flat and rigid section.

IV.7 Protection device according to any preceding claim, wherein the cover part is divided into three parallel sections that are respectively joined by parallel hinges, and which are foldable into a triangular prism shape.


16.1 Claim IV.7 defines that the cover part is divided along its longer dimension into three parallel sections that are respectively joined by parallel hinges, and which are foldable into a triangular prism shape

    False: Claim IV.7 misses “along its longer dimension”

16.2 Claim IV.3 meets the requirements of Art. 84 EPC

    False: Reference to a figure; the flat base could have been defined in words (see, e.g., claims IV.4-6) which means that a definition by reference to a figure is certainly not allowable.

16.3 A protection device comprising a receiving part for receiving a tablet computer and a cover part having two hinges between the cover part and receiving part falls within the scope of claim IV.2

    True: ‘comprises a hinge’ does not exclude the presence of two or more hinges

16.4 Claim IV.4 lacks clarity because of an incorrect dependency

    False: No problem with the dependency; the receiving part is already defined in claim 1.

 

17.1 D1 destroys the novelty of the subject-matter of claim IV.4

    True: D1 discloses a frame made of elastically deformable material as the receiving part (‘tablet computer held tightly therein’) and a cover part in the form of the folded flaps.

17.2 D1 destroys the novelty of the subject-matter of claim IV.2

    True: The bottom part of the shipping box can be considered as a receiving part (“flat base and four walls for receiving the tablet computer”) and the flaps as the cover part; the flaps are attached to the walls via hinges.

17.3 D1 destroys the novelty of the subject-matter of claim IV.6

    True: ‘according to any’ = OR claim. D1 is at least novelty destroying for claim IV.6+IV.1 (the broadest alternative) and thus novelty destroying for the claim. Namely, D1’s flat base is rigid and flat.

17.4 D1 destroys the novelty of the subject-matter of claim IV.7

    False: D1’s flaps (or other features) do not seem to be arranged as three joined parallel sections.

 

18.1 The subject-matter of claim IV.4 is novel over D2

    False: See [002] of D2: “Both the flat base and the peripheral wall deform elastically for receiving and securing the tablet computer in the receiving part”; this anticipates at least the combination of claim IV.4 + IV.1, i.e., the broadest claimed alternative.

18.2 The subject-matter of claim IV.7 is novel over D2

    False: See [004] of D2; while it is not disclosed that the hinges are parallel, this seems to be implied by the sections being arranged in parallel and joined by hinges.

     (*) True can also be argued: D2 [004] refers to Fig.3 in D2, but the figure shows 4 sections folded into a triangular prism whereas the description says three. You may consider this an error in a prior art document, and use GL G-IV, 9 to conclude that either the figure is correct and the text wrong (such that D2 shows a cover split into 4 sections rather than the claimed 3 so that the claim is new) or that multiple corrections of [004] and Fig. 3 are possible (such that D2 must not be taken into account, so that the claim is new).  See comments posted to this blog. [Added 5 March 2021]

     Note that the claim is limited to a cover part that is divided into exactly 3 sections (i.e., consists of 3 sections) rather than that it has 3 or more (comprises 3 sections). ("IV.7: Protection device according to any preceding claim, wherein the cover part is divided into three parallel sections that are respectively joined by parallel hinges, and which are foldable into a triangular prism shape.") 

     [Update 2 April 2021 8:45:]
     The statement was not neutralized in the Examiner's Report. Only False was considered correct.
     The Examiner's report argumentation is discussed in the comments to our "Results" blog (here).
     Please feel invited to contribute to the discussion there.


18.3 The subject-matter of claim IV.6 is novel over D2

    False: See [004] of D2; the cover part comprises three flat and rigid sections and thus “a” flat and rigid section as required by the claim.

18.4 The subject-matter of claim IV.5 is novel over D2

    False: D2’s entire peripheral wall is made of silicone and deforms elastically, and thus also its four corners (Fig. 3 shows that the peripheral wall has four corners).

 

19.1 Under Article 123(2) EPC, there is basis for amending claim IV.1 of the originally filed application as follows:

Protection device comprising a receiving part for receiving a tablet computer and a cover part, wherein the receiving part comprises a flat base and four elastically deformable corners for securing the tablet computer within the receiving part, and wherein the cover part comprises a flat and rigid section

    True: Supported by the combination of claims VI.6, 5, 4, 3 and 1, while deleting the optional ‘preferably as shown in Fig. 1’ part

19.2 Under Article 123(2) EPC, there is basis for amending claim IV.1 of the originally filed application as follows:

Protection device comprising a receiving part for receiving a tablet computer and a cover part, wherein the receiving part comprises a flat base and four elastically deformable corners for securing the tablet computer within the receiving part, and wherein the cover part is flat

    False: While the first embodiment has a flat cover part, it is also rigid, which is now omitted from the claim without this being directly and unambiguously derivable from the application.

19.3 Under Article 123(2) EPC, there is basis for amending claim IV.1 of the originally filed application as follows:

Protection device comprising a receiving part for receiving a tablet computer and a cover part, wherein the cover part is divided into three parallel sections that are respectively joined by parallel hinges, and is foldable into a triangular prism shape, each section being flat and rigid, and wherein the hinges and the sections are made from the same material such that the hinges are formed by folds of the cover part

    True: Seems to be supported by the second embodiment (with the wording ‘parallel’ in conjunction with the ‘sections’ coming from claim IV.7).

     [Update 2 April 2021 8:45:]

     (*) False can also be argued - see K's comment on our "Results" blog (here), who indicates:

"If "a receiving part being a flat base and having four elastically deformable corners" applies to all embodiments of the invention, then taking the receving part from [005] requires to take it in the way it is disclosed as applying to all embodiments of the invention. Therefore, it is necessary to include the flat base and the four elastically deformable corners in the claim. As this was not done in the amended claim [and those features are necessary to hold the table computer in the receiving part], the statement is FALSE."

     The statement was not neutralized in the Examiner's Report. Only True was considered correct.
     The 
Examiner's Report (where it is statement 19.4)provides:

"TRUE - This amendment is based on the combination of IV.1, IV.7 and [005] of the description of the application. The invention is defined in paragraph [003] of the description of the application to have the receiving part being a flat base and having four elastically deformable corners. Thus, this receiving part applies to all embodiments of the invention. Therefore, it is not necessary to include the flat base and the four elastically deformable corners in the claim. Thus, this amendment complies with Article 123(2) EPC."

     The Examiner's report argumentation is discussed in the comments to our "Results" blog (here).
     Please feel invited to contribute to the discussion there.

19.4 Under Article 123(2) EPC, there is basis for amending claim IV.1 of the originally filed application as follows:

Protection device comprising a receiving part for receiving a tablet computer and a cover part, and further comprising a hinge between the receiving part and the cover part, the receiving part comprising a base, the base being flat

    True: This corresponds to the combination of claims IV.3, 2 and 1 while omitting the “preferably as shown in Fig. 1” part.

 

QUESTION  20

For this question, assume that the following Claim V.1 was the original claim filed in the client patent application.

V.1 Protection device comprising a receiving part for receiving a tablet computer and a cover part, wherein the receiving part comprises a flat and rigid plastic base and four corners for securing the tablet computer within the receiving part, characterised in that the four corners are elastically deformable and the four corners extend from the peripheral edge of the base and the four corners are made of silicone having a greater thickness than the base.


20.1 A valid argument as to why D2 may be considered the closest prior art is that it discloses a base made from the same material as the corners

    False: Claim V.1 actually defines that the base is made of a rigid clear plastic and that the corners are made of silicone, and thus not that the base is made from the same material as the corners. The reasoning is thus based on a feature not part of claim V.1

20.2 A possible technical effect of the invention defined in claim V.1 with respect to D2 is that it allows better protection of the screen of the tablet computer

    False: D2 states that its thickness of the whole receiving part may be increased uniformly. Only having thicker corners seems to rather reduce cost/address environmental concerns, see [003]. In fact, according to [003], the protection may be slightly decreased.

20.3 For the purpose of assessing inventive step of claim V.1, a valid argument is that D1 does not relate to the technical field of protection devices

    False: A shipping box is arguably also a protection device.

20.4 Claim V.1 is in the correct two-part form with respect to the disclosure of D2

    False: For at least the reason that D2 does not disclose a rigid plastic base; this should be in the characterizing portion.


Our Pre-Exam 2021 blog is composed of three separate blogs:

  • first impressions with your comments as to first impressions, the Pre-Exam 2021 as a whole, the e-EQE platform, etc
  • legal part with our answers to the legal questions
  • claims part with our answers to the claims analysis questions

We look forward to your comments!

Comments are welcome in any official EPO language, not just English. So, comments in German and French are also very welcome!

Please do not post your comments anonymously - it is allowed, but it makes responding more difficult and rather clumsy ("Dear Mr/Mrs/Ms Anonymous of 02-03-2021 22:23"), whereas using your real name or a nickname is more personal, more interesting and makes a more attractive conversation. You do not need to log in or make an account - it is OK to just put your (nick) name at the end of your post.

Thanks!

(c) DeltaPatents

Comments

  1. Question 19.3)
    I disagree with the proposed solution for question 19.3:

    “Under Article 123(2) EPC, there is basis for amending claim IV.1 of the originally filed application as follows:
    Protection device comprising a receiving part for receiving a tablet computer and a cover part, wherein the cover part is divided into three parallel sections that are respectively joined by parallel hinges, and is foldable into a triangular prism shape, each section being flat and rigid, and wherein the hinges and the sections are made from the same material such that the hinges are formed by folds of the cover part."

    Potential basis in the claims:

    Protection device comprising a receiving part for receiving a tablet computer and a cover part,
    (claim IV.1)
    wherein the cover part is divided into three parallel sections that are respectively joined by parallel hinges, and is foldable into a triangular prism shape,
    (claim IV.7)
    each section being flat and rigid, and wherein the hinges and the sections are made from the same material such that the hinges are formed by folds of the cover part.
    No basis in original claims, since merely in claim IV. 6 “comprising a flat and rigid section”; nothing about material and so on.



    Potential basis in embodiment 2:

    Hence, basis could only be embodiment 2 in [005].
    However, embodiment 2 also discloses a receiving part comprising a flat base and four elastically deformable corners.
    Amended claim IV.1 as above appears to merely specify a generic receiving part.

    Therefore, I would argue that the solution to question 19.3 is False based on the amendment being an unallowable intermediate generalisation.

    ReplyDelete
    Replies
    1. Also arguable, I thought, that (at least in embodiment 2) the parallel hinges between sections of the cover are parallel to the hinge between the cover part and the receiving part, not only parallel to each other. I would agree that this was intermediate generalisation.

      Delete
    2. Good point, we missed yesterday that embodiment 2 is more specific with respect to the proposed amended claim in that the claim fails to include the four elastically deformable corners of the second embodiment. The question indeed is whether omission of this feature satisfies Art. 123(2). However, taking the intermediate generalization test as an aid, the answer is not clear cut (at least to us). For example, one could still argue that the application as filed suggests embodiments without four deformable corners (i.e., the originally filed claims IV.1 and IV.4), and thus that the four deformable corners are not an essential feature of the invention. Moreover, when applying the intermediate generalization test, one could argue that the four corners are not "inextricably linked" to the cover and its material/hinges, and perhaps are even not "related" to the cover.

      Delete
    3. I came to the same conclusion, because the amended claim does not mention the receiving part comprising a flat base and four elastically deformable corners extending from the peripheral edge of the base, which are not optional features. Par. 003 also specifies the features of the receiving part as part of the invention. The overall disclosure does not justify the generalising isolation of the features of embodiment 2 (which are part of this amended claim) without the features of the receiving part.

      Delete
    4. I am the anonymous from above. As in the real world "inextricably linked" is at least for me a quite subjective argument if no clear statement in this direction is provided. If you ask 5 people you get 10 different opinions. :) Maybe I missed the clear pointer. In previous exams they often had a statement that "feature A" is totally independent from "feature B" and can be combined with it in all forms and embodiments. Well, let's see what they wanted to test with this question. I presume they had some concept in mind.

      Delete
    5. In light of the patent application itself and the prior art it is clear that the four elastically deformable corners are an essential feature, as using the four elastically deformable corners allows for reduced cost and environmental friendliness, while only slightly reducing the protection.

      Delete
  2. 11.3 is repeated in 11.4
    12.2 is repeated in 12.3
    2 solutions missing.

    ReplyDelete
  3. Regarding Q15.4: [011] states that "component X provides improved resistance to colour changes as a result of a change in temp". Thus, a colour change resistant to temp change is already achieved by X without the provision that X has to be alpha. Since D2 also contains X, the technical effect of resistance to colour change is already achieved by D2 and this can therefore not be the objective technical problem. In my opinion, the objective technical problem is "how to provide improved resistance to colour change", and answer to the question is therefore false. Any other views?

    ReplyDelete
    Replies
    1. There is no provision that the prior art must not also solve the same objective technical problem. As long as the claimed solution to the objective technical problem is new and inventive, it doesn't matter that the prior art solves the same problem.

      Delete
    2. I completely agree with Charlotte. The objective technical problem must be based on the distinguishing features. The distinguishing features are that component alpha is used (and not beta or gamma). In my opinion, the objective technical problem can therefore not be "the provision of a photochromic lens, the colour change of which is resistant to temperature changes", because this does not refer to the difference, since all the three components (alpha, beta and gamma) contribute to this problem. The objective technical problem could be formulated for example as "how to improve resistance to colour changes as a result of a change in temperature".

      Delete
    3. Hi Anon:
      "The objective technical problem must be based on the distinguishing feature"

      It is. The distinguishing feature is alpha. A technical effect of alpha is a photochromic lens, the colour change of which is resistant to temperature changes.

      Just because the prior art also solves this problem doesn't make it not a valid objective technical problem for PS.

      I do agree that the objective technical problem could have been formulated much better for the question, but the statement presented with is still true.

      Delete
    4. Also, the technical effect doesn't have to be new.

      Delete
    5. Hi Beans, I disagree with your arguments. GL G VII 5.2 clarifies the following: "In the context of the problem-solution approach, the technical problem means the aim and task of modifying or adapting the closest prior art to provide the technical effects that the invention provides over the closest prior art. The technical problem thus defined is often referred to as the "objective technical problem"."
      Thus, in this context, one should identify the technical effects that the invention provides over D2. If there is no technical effect over D2, then the objective technical problem should have been "to provide an alternative". In any case, the presented statement is false.
      Also, similar questions have been tested in previous exams (e.g. 2018 Q19.4).

      Delete
    6. Am I the only guy think that the objective technical problem in the question actually has a pointer to the solution, and therefore, it is false?

      Delete
    7. That is what my thinking is too!

      Delete
    8. @Monk - I thought similar and that the objective techinical problem should have been framed in the context of protection for better protection from UV light, as referenced in para [003] as the reference to colour changes is a pointer to the solution employed by the invention. I also agree with Charlotte - in this case, the distinguishing feature is use of alpha and the technical effect of this is that the resistance to colour change *increases* compared to using beta and gamma in the prior art. This is all without the initial problem raised by Nico that the claim is to a glass composition FOR a photochromic lens and therefore the technical problem should be formulated in reference to the glass composition and not the photochromic lens

      Delete
    9. When I was preparing for the exam I was also struggling sometimes with inventive step and intermediate generalisation in the claim analysis part. I found it tricky to find a good balance and not to overthink the questions.

      In this case I don't agree with monk and shanon. Where exactly do you see a pointer?

      Delete
    10. @Billygoat - I can also follow the Charlotte's reasoning, and indeed agree that the problem should also be directed to the glass composition not to the photochromic lens.
      Nevertheless, during the exam, the pointer to solution in the question is so strong that it stopped me from checking further whether it should be the glass composition or the photochromic lens ...

      Delete
    11. @Anonymous2 March 2021 at 12:58
      The pointer is "the colour change of which is resistant to temperature changes", which is in the technical effect and I don't think such technical effect should be put in the O.T.P.

      Delete
    12. The solution is the use of alpha as X. This isn't in the formulation of the objective technical problem?

      I marked it as true, though I can follow the reasoning that it should be an improved resistance.

      Delete
    13. The solution (compared to D2) is to use compound alpha, a distinguishing characteristic of which when compared to beta and gamma is its resistance to colour changes as a result of temperature. The technical problem as formulated literally says to go and use a compound with those properties. The pointer is, from my reading of the question, really quite strong and, like Monk, I sort of ignored the other issues with the formulation of the OTP because of this as I felt they were relatively minor in comparison

      Delete
    14. I agree with Charlotte. In accordance with the guidelines, the problem is posed with respect to D2 - and the inventive solution solves this problem. The problem should therefore at least include 'alternative means for..' or 'improved resistance..'.

      Delete
    15. Hi all, in my view the question at hand is not whether or not the solution is inventive, but what the technical problems is in the view of D2. If D2 already solves the problem of providing resistance to temperature changes, the technical problem would then be to find an improvement (compared to D2) or alternative solution to the one provided in D2. I.e. the solution of the invention is still inventive, but the problem that should be used in the problem solution approach is different.

      Delete
    16. I also thought that the above stated objective technical problem gives pointers to the solution, which is not allowable. The objective technical problem should have been formulated as providing an improved glass composition for use as a photochromic lens.
      I think the answer should have been FALSE.

      Delete
    17. @Billygoat @Peny, after reading DCF's comment in the following regarding OTP has pointer to the solution. Now I can understand and does not think the OTP actually points to the solution. The reason what I have learnt about is:

      you have to forget about the compound alpha having the concerned technical effect, because the effect is described as "surprisingly", i.e., the effect of alpha alone is not commonly known, but only found in the context of invention. Therefore, the skilled person starting from the OTP, have to think a solution WITHOUT knowing alpha has this effect (the hindsight knowledge). In this case, the skilled person would have no clue, or cannot find any point to the alpha itself from this OTP (remember you have to forget about knowing alpha has this surprising effect!).

      Nevertheless, I can follow the argument that the OTP should be "further improve" or "enhance"; or the OTP should be directed to the glass composition, not to lens, and therefore is wrong for these grounds.

      Delete
    18. I agree that the objective technical problem as formulated in the question gave pointers to the solution. As also stated in the answer for Q15.4, this must be the technical effect of alpha, not the technical problem.

      Delete
    19. @Xi, this technical effect of alpha is the hindsight knowledge, i.e., not known before the invention.

      For example, I found that eating a specific amount of sugar everyday has the surprising effect that I can become smart. Then the OTP is how to become smart. Does this OTP leads to you think of sugar? Apparently not.

      Delete
    20. In this instance any 'resistance' is merely a reduced sensitivity. i.e. to require resistance is implicitly to require more of it, and so less of the sensitivity. 'Improved' would be redundant.

      Anna

      Delete
    21. I disagree Anna, the resistance is certainly quantifiable. 'Improved' is not at all redundant.

      Delete
    22. Charlotte wrote:
      "the technical effect of resistance to colour change is already achieved by D2 and this can therefore not be the objective technical problem"

      The objective technical problem CAN be a known problem.

      Example: the objective technical problem can be "how to increasee the speed of a car"

      This problem has certainly been tackled already in many ways.

      But if you can solve it in a not-yet-known way, e.g., by a new aerodynamical shape, you are inventive.

      It may also be that the closest prior art and the claim get you to this OTP, but that the "invention" solves it in the same way as a second prior art document (for which there was a reason to combine it with the closest prior art), so that you are not inventive. The latter does however not disqualify the formulation of the OTP: that is just determined by the effect of the distinguishing feature (difference between claimed invention and CPA).

      Delete
  4. I was thinking the same as Charlotte and that's why my answer was false

    ReplyDelete
    Replies
    1. You and Charlotte are wrong. Juan is right, Problem may be well known! As long as solution provided by the claim is not known, claim is inventive.

      Matt

      Delete
    2. Hi Juan and Matt, could you please elaborate on your reasoning? The objective technical problem should be based on the technical effect of the distinguishing feature. Here, the distinguishing feature is alpha. The technical effect can only be that it provides improved resistance over X, since X as such already provides resistance (see Par. [011]).

      Please explain how the claim can be inventive if the objective technical problem is already solved by the closest prior art?

      Delete
    3. Hi Charlotte:

      "Please explain how the claim can be inventive if the objective technical problem is already solved by the closest prior art?"

      As an example: would you tell a client that their new and inventive solution to a known technical problem cannot be patented, because the prior art solves the same problem (by a different means)?

      Delete
    4. I completely agree with Charlotte.

      Juan, I don't think that is Charlotte's point. There is a difference between a 'technical problem' and the 'objective technical problem' *in the framework of the EPO problem-solution approach*. A technical problem may of course have multiple inventive solutions (which is what you hint to with the client example), but the 'objective technical problem' is posed with respect to D2 - as such, the term 'alternative means' or 'improved' should be included for consistency with the guidelines.

      Delete
    5. So glad to see so many people discussing this Q. I also put false, for exactly the same reasons as Charlotte and the other few above who mention that the problem of resistance to temperature change was already solved in D2 by the use of beta or gamma, and so could only be an alternative or improved resistance. I can't follow Juan's reasoning: what you should or shouldn't tell a client isn't going to have any bearing on the tests that the EPO use to assess inventiveness...

      Delete
    6. I agree with Charlotte, Tom et al., for answering "T", the statement should have been formulated "... the provision of a photochromic lens, the colour change of which has IMPROVED resistance to temperature changes." Beta and Gamma of D2 also provide resistance to temperature changes, therefore I answered "F".

      Delete
    7. I disagree with Charlotte. D2 does not disclose the use of Component X in order to solve the problem of temperature sensitivity. In fact neither of the A54(2) documents disclose the problem of temperature sensitivity which is disclosed only in the 54(3) art and the present invention. Given the artificially limited information provided in the exam it is fine to state the OTP with respect to D2 as being resistance to temperature change.

      I put true.

      Anna

      Delete
    8. Anna, the question of whether the problem itself is disclosed is not relevant.

      I agree with Charlotte for all the reasons above.

      Delete
    9. Hi Anonymous at 20:16. Spotting the problem can be inventive. https://www.epo.org/law-practice/legal-texts/html/caselaw/2019/e/clr_i_d_9_11.htm

      Anna

      Delete
    10. @Anonymous 3 March 2021 at 23:32: there is no such "problem invention" in this exam.

      For inventive step: need to sue the problem-solution as it is detailed in the Guidelines. We summarize it more-or-less as the below in our courses:

      - you need to determine the closest prior art (by comparing the claimed invention = claim as a whole to the various prior arts: is there any in the same field? If more then one, which is closest in aim/purpose? If then still two likely candidates, which is structurally/functionally the most similar so that the effort of modifying that to come to the invention is lowest);
      - then you determine the distinguishing feature between the claim and the closest prior art (what makes the claim novel);
      - then check in your own application which effect(s) are obtained from that feature;
      - then formulate the objective technical problem:
      -- if effect not yet known from the CPA:
      how to obtain that effect;
      -- if effect already known from the CPA (but obtained by a different feature):
      how to provide an alternative

      Now that you have your OTP, you can check whether there is an incentive to consider other prior art which may present you with a solution to the OTP. If there is such incentive, and the solution is given by the same feature, then (generally) not inventive.
      If there is no such incentive, or there is but the solution is given by a different feature, then (generally) inventive.

      Delete
  5. I agree with Charlotte and her reasoning. According to the problem-solution approach which is to be applied here an objective technical problem is formulated based on a "difference" between the claimed invention and the closest prior art (i. e. D2) (see G-VII, 5.2). / Irina

    ReplyDelete
    Replies
    1. Not based on a different effect, but derived from a distinguishing feature

      Matt

      Delete
    2. Exactly. And as Charlotte correctly pointed out, the techcnail effect of the distinguishing feature is not a temperature-resistance of the lens but an INCREASED temperature resistance. If there was no increase over D2, than both documents would disclose temperature resistant lenses --> no technical effect of distinguishing feature --> problem to be solved is only providing an alternative.

      Delete
    3. @DCF: maybe I am misinterpreting your words - are you saying that an alternative solution to known problem is not patentable?

      The solution has to be new and inventive, not the objective technical problem.

      "the techcnail effect of the distinguishing feature is not a temperature-resistance of the lens but an INCREASED temperature resistance". Why not both? A feature can have more than one technical effect.

      Delete
    4. Indeed, a better formulation of the statement would have been 'more resistant'.

      However, when only considering the difference between the invention as claimed and D2, the invention as claimed may indeed be considered to be 'resistant to colour changes' (i.e., by taking D2 as a baseline/starting point).

      By the way, the reference to 'photochromic lens' seems to be an even bigger ambiguity in this question...

      Delete
    5. Isn't the objective technical problem hinting to the technical solution here? I thought this was not allowed and hence the answer to this q is false?

      Delete
    6. In my opinion, in this case it should be formulated differently, for example by "how to provide an alternative way of provision of a photochromic lens, the colour change of which is resistant to temperature changes". If it is formulated as it is, the solution is already provided in D2.

      Delete
    7. I agree with Ash here - I thought the OTP was pointing towards the solution and thought it should be more to do with improved protection from UV light, as referenced by [003]. I also agree with Nico about the ambiguity related to the photochromic lens vs glass composition

      Delete
    8. I also thought that the otp pointed to the solution and therefore answered "false", but coming to think of it Im afraid it is pretty clearly the wrong reasoning.
      It is important to clearly distinguish problem, effect and solution.
      effect: (increased) temperature resistance
      problem providing (increased) temperature resistance
      solution: employing compound alpha.

      So the problem somewhat "points" to the effect because they are so similar, but NOT towards the solution, i.e. compound alpha.
      In fact, it would be quite difficult here to make a problem that would hint at employing alpha.
      More care has to be taken in other cases where "solution" and effect are quite similar.
      e.g. when in an application the difference is that the inventive composition further comprises a sweetener, the effect is to make the composition sweeter. There it is very important (and sometimes difficult) to formulate the problem in a way to NOT hint at the solution.
      Maybe the problem could be to make the taste more pleasant, but the problem obviously may not be how to make it sweeter.

      Delete
    9. @DCF, thanks for your detailed input. I completely followed your point and can understand now the concerned otp does not point to the solution, i.e., to the compound alpha.

      Delete
    10. I agree with Charlotte since the invention clearly defined in paragraph [0011] that component X provides improved resistance to colour change as a result of a change in temperature. Preferably, component X is selected from compound alpha,compound beta and compound gamma [...]. To my understanding, compound beta and compound gamma also provides resistance to colour change as a result of a change in temperature. Accordingly, the problem should be formulated as the provision of a photochromic lens, the colour change of which has an improved resistance to temperature changes.

      Delete
    11. Thx DCF for your nice inventive step argumentation. I agree with it.

      Delete
  6. Regarding 12.3: "Claim I-8 is unclear due to the use of the term low".
    I think the above phrasing is unclear and ambiguous.

    I agree that claim I-8 is unclear.
    I agree that the use of the term "low" renders it unclear.
    I disagree that claim I-8 is unclear (only) due to the use of the term low.
    I agree that claim I-8 is unclear AT LEAST due to the use of the term low.
    Another term which renders claim I-8 unclear is "high".

    Irina

    ReplyDelete
    Replies
    1. I actually don't think that claim I-8 is unclear (though I do think that may well be the answer they want to hear, especially given that, in my understanding, the "EQE skilled person" is different from the "EPC skilled person" in that they possess only the general knowledge mentioned in the provided documents).

      "High" and "low" are relative terms that are potentially unclear, but they may be clear in the context of the overall disclosure. GL F-IV, 4.6 gives some pointers for interpreting such terms and assessing their clarity (admittedly I haven't looked at the case law).

      Here, I would argue that

      a) "high" and "low" are to be interpreted in relation to one another (cf. F-IV 4.6.2),

      b) specific examples for "high" are given in the description,

      c) "low" is not the only (or even a) distinguishing feature with respect to the prior art , therefore the reasoning regarding a generally accepted meaning does not apply (F-IV 4.6.1),

      d) it is directly and unambiguously derivable from the disclosure that "high temperature" refers to a glass melt and "low temperature" refers to a solidified glass, and

      e) par. 6 of the application even mentions that glass has a melting temperature and the skilled person will understand that, in view of point a) and e), "high" and "low" must refer to temperatures above and below the melting point.

      In conclusion, the use of the term "low" does not render the claim unclear.

      Delete
    2. Hi MF,

      can you tell me which temperature ranges correspond to "high" and "low"?

      Matt

      Delete
    3. I reasoned the same as MF and also came to the conclusion that the term "low" is not unclear. Indeed, it is derivable from the disclosure that "high temp" refers to glass melt and "low temp" to solidified glass. But also agree with MF that this is probably not the correct answer according to the committee.

      Delete
    4. I also agree with MF that the term "low" would be clear only if the temperature ranges are specified. Temperature at which glass melt or solidify is to generic and also depend from other parameters, like particular glass composition, as explained here:https://hypertextbook.com/facts/2002/SaiLee.shtml
      They also say that "glass doesn't actually melt per se. Rather it goes through a glass phase transition."

      Delete
    5. Thank you for for presenting your opinions. I am still, however, of opinion that "low" and "high" are not clear. I do agree though with MF and Charlotte, that it may be deducted from the description that "low" means a temperature below the melting point. I think it is important to discuss the meaning behind the clarity requirements. There are only claims, not description, that will be published in the EPO official languages. Therefore, it makes sense to strive for some real clarity: "the meaning of the terms of a claim must, as far as possible, be clear for the person skilled in the art from the wording of the claim alone" (see F-IV-4.1 and 4.2). If we turn to claim I-8 and read it in isolation, I believe it is unlikely that a person skilled in the art would be able to figure out that "high temperature" and "low temperature" have any relation to the melting point.

      Irina

      Delete
    6. You may argue that "low" and "high" can be considered relative terms, but not more than that: "low" is lower than "high", but what temperature ranges they refer to is not defined.
      If you just read the claim, there is no reason to assume that "high" cannot be 100 or 500 °C.

      You may check whether the the description tells you that the skilled person has a general knowledge of what is "low" and "high" - GL F-IV, 4.6.1. But it does not have any such common general knowledge information.
      On the contrary, [016] suggest that "high" is much higher:
      "Preferably, the silicon dioxide is heated to about 1500 °C, about 1600 °C, about 1700 °C or about 1800 °C. Most preferably, the silicon dioxide is heated to 1675 °C"
      So: is "high" meant to indicate 1500 - 1675 °C? But it said preferably, so lower or higher may also be fine, maybe 1200 - 2300 °C? Or 400 - 4000 °C?
      As this is not clear, the claim is not clear.

      For "low", there is no temperature indication at all? is that room temperature? Or below zero? Or below 200? Or below 1575? Not clear.

      Counter argument:

      for clarity, only needed that the claim wording is understood, not that the invention words over the whole range as understood from the wording - that is Art.83.

      With that interpretation, heating to high, forming while heated, cooling to low, is clear: it defines that you do the forming at an higher temperature.

      Delete
    7. When discussing further on the matter to try to get more convincing arguments why the statement is/is not clear, Nico and I noted that the terms "low" and "high" may be considered relative to each other, but that even then, the claim is not clear to to the term "low":

      F-IV 4.6.2 Clarity objections:
      "Relative or similar terms such as "thin", "wide" or "strong" constitute a potentially unclear element due to the fact that their meaning may change depending on the context. For these terms to be allowed, their meaning must be clear in the context of the whole disclosure of the application or patent."

      Whole the application does provide preferred ranges for a "high" temperature, there is not a single range or value mentioned of what would be a "low temperature".

      So, one cannot conclude whether "low" means less than room temperature, or a bit less than "high" but still several hundreds degD, or ...

      We thus concluded that the "Claim I-8 is unclear due to the use of the term low" (and not so much due to the term high), i.e., that the statement is TRUE.

      Delete
    8. @Roel 11:10

      I followed this exact process in the exam - high is disclosed in the application as a range of temperature, whereas low is not mentioned at all. I also concluded true.

      Delete
  7. Regarding 15.3: "D2 can be considered to be the closest prior art for claim III-3 because it relates to the same technical field, i.e. the field of photochromic lenses". I think the correct answer is FALSE.

    I do not think that D2 can be considered the closest prior art just because it relates to the same technical field. There is a test for determining the closest prior art as outlined in G-VII, 5.1, i. e. "The closest prior art is that which in one single reference discloses the combination of features which constitutes the most promising starting point for a development leading to the invention. In selecting the closest prior art, the first consideration is that it must be directed to a similar purpose or effect as the invention or at least belong to the same or a closely related technical field as the claimed invention. In practice, the closest prior art is generally that which corresponds to a similar use and requires the minimum of structural and functional modifications to arrive at the claimed invention (see T 606/89)."

    Irina


    ReplyDelete
    Replies
    1. D1 and D2 both disclose all the features of the claim in question, so from a "combination of features" perspective there is no better starting point. Thus, the actual determination that D2 is closest prior art comes down to it relating to the same technical field.

      Delete
    2. I also think that is FALSE.
      The purpose is more important as the technical field.
      Moreover, i interpret the technical field as being "glass composition"

      Delete
    3. I was actually torn between True and false for this one for both of the reasons above but ended up choosing True after flicking through some of the answers from previous years where technical field for determining the closest prior art was used and considered relevant. Even if it is not the most relevant factor, it's still a relevant factor that makes D2 a more promising starting point in this circumstances, so I thought it safer to put false. I think if there were two documents in a similar field, it would certainly have been False, but since the only two documents were one in a similar field, and another in an unrelated one, I think it was a valid statement.

      Delete
    4. Can someone please explain, if the technical field is photochromic lenses and the technical effect is "resistance to color changes as a result to temperature changes", which is the purpose/use of the invention?
      Because I thought "purpose/use"= "photochromic lenses", and field = "glass composition".

      Delete
    5. I also thought that 15.3 is FALSE because of the technical field. I thought technical field is related to glass composition not to photochromic lenses

      Delete
    6. I also strongly believe that the correct answer is false.
      All three documents disclose inventions related to glass compositions. D1 even specifies that it relates to glass composition suitable for use (therefore not limited to) as glass bottle. Therefore, one should not assume in this case that D1 is a more remote technical field.
      I believe that if D1 was disclosing a composition similar to that of the application, it would be considered the closest prior art document regardless of whether the invention is suitable for use as a glass bottle.

      Delete
    7. As you may have noted, we placed an * to 15.3 and 15.4 when we posted our answers, for the reason that Gamze also raises just above.

      We had the same doubt as Gamze: is the technical field glass compositions (not necessarily limited to those suitable for photochromic lenses), glass compositions suitable for photochromic lenses, or photochromic lenses? or is that all the same?

      [001] of the application says "The present invention relates to a glass composition for use as a photochromic lens". That suggests that the glass composition is shaped to form a photochromic lens, i.e., that it is a lens: not just some powder or so, but a tangible device (lens). That goes against my understanding of what a composition is: it is the mere chemical stuff, e.g. provided as a powder. In medical claims, a composition is the active ingredient of the pill, not the pill itself (which also includes chalk and other stuff).
      Same in [004].
      The problem arises from the wording "for use AS lens", rather than "for use in manufacturing a lens""or "for manufacturing a lens".

      But [015] clearly defines that the composition and the lens are different entities: "The invention also provides a process for manufacturing a photochromic lens FROM the glass composition of the invention": it makes the lens from the composition.
      More specifically, A, M and C in the method are components of a composition according to the invention, and these are mixed/heated/added (to make up the composition) and then formed into a lens, that is then cooled.
      This is in line with my interpretation of the word composition: it is a mixture of various components, but not yet shaped/formed into a device.

      As claim II.3 is not directed to a lens, but to a glass composition (for use as a lens), the closest prior art must be a glass composition that is suitable "for use as" a lens, i.e., -with the interpretation above: composition is not the same as device- glass composition that is suitable "for manufacturing" a lens.
      Then, 15.3 is FALSE.

      Same for 15.4: the OTP should be “the provision of a glass composition for use as a photochromic lens” = “the provision of a glass composition for manufacturing a photochromic lens”.
      Thus, 15.4 is FALSE.

      Only if the glass composition would be equated to being a photochromic lens then TRUE.

      Note that statement 15.1 seems to be TRUE in either interpretation.
      This may have as a side-effect that the order in which the statements are presented may have an effect as to how a candidate may interpretation the "for use as" and the statements.

      Delete
  8. Does anyone know what happens if you appeal and the examiner agrees the answer could have been true or false? As in do only the people that appeal get marks for that question or does every candidate get awarded the marks?

    ReplyDelete
    Replies
    1. Hi Shanon, if the question is deemed unclear, then the examiner awards marks for both true and false (see the examiners report of the 2018 paper).

      Delete
    2. If they decide before marking then everyone gets a mark irrespective of answer. However, if it is in response to a post-results appeal process then only the appellant is given the extra marks.

      Anna

      Delete
    3. I agree with Anna. It has happened that after an appeal all candidates were given extra marks but I think that is an exception rather than the rule. But if the examiners decided that a question is unclear before the give the marks, everyone will be given the extra mark. I have also heard that the examiners read blogs such as the DP blog and therefore it is important to raise issues such as unclear/ambiguous questions on forums such as the DP blog. I am not entirely sure if this is true so would be good if Nico or Stefanie could comment on this.

      Delete
    4. The Committee and the Examination Board take into account any information that comes to their attention to check whether statements need to be neutralized or not.

      In the last few years, neutralizations took place already when the official marking was done. Presumably, statistical analysis of answers, discussions on the blog, information send to the EQE helpdesk, ... were used to check and conclude on where neutralization was needed. Only few appeals were filed in the last few years (as far as we know), now that this neutralization is part of the procedure - in the first few years, neutralizations happened moer often as a result of an appeal (sometimes for the individual candidate that appealed, sometimes for all candidates - but note that every PASS/FAIL decision is an individual decision, so you would need to appeal yourself if you want to be sure that an appeal may change your result; there is no legal provision that entitles you to a change of your EQE decision if someone else appealed but you did not!)

      Delete
    5. Thanks for the clarification Roel, and for all the good work you and your team or doing with this blog!

      Delete
    6. In other words:
      if you want to be efficient and if you want early certainty, post your arguments on this blog! If it is well reasoned and you have a true point, it will be considered.

      Note that an appeal an take long. E.g., the most recent EQE 2019 Pre-Exam appeal was D 3/19 which was decided on 01.07.2020 while it was filed in April 2019 against the Examination Board's decision of 18 March 2019 (awarding the appellant the grade "fail" in the pre-examination of the European qualifying examination 2019, her answer paper having been given a score of 68). So, the decision was too late for a 2020 main exam enrolment.

      Also note that the Disciplinary Board only has a limited competence as what they can review - a pure interpretation matter is beyond their power. So, if you need to discuss the details of the question and/or the answer, raising that now, prior to the PASS/FAIL decision, is more appropriate.

      Delete
  9. Regarding question 14.1 Claim II-3 is allowable under Article 123(2) EPC, marked as "true":

    True: True, claim II-3 corresponds to claim I-1 but with an allowable disclaimer (‘not zinc oxide’) according to G2/10.

    Not being Zinc Oxide does not equates being one of the other two possible materials, it was a exhaustive list. So the disclaimer represents an addition of subject matter and thus no allowable.

    ReplyDelete
    Replies
    1. If I remember correctly, the list is introduced as "preferably" and thus not exhaustive.

      Delete
    2. A disclaimer may be allowable in order to restore novelty by delimiting a claim against state of the art under Art. 54(3) and document D3 discloses zinc oxide and D3 is a prior art document under Art. 54(3)

      Delete
    3. Delimiting a claim with an undisclosed disclaimer serves to make the application patentable over a 54(3) document. This would not be the case here, because claim 1 still is not novel. Therefore, I dont think the pointless exercise of making claim 3 novel is allowable.

      But I also dont know why the disclosed disclaimer would be allowed. I know the criterion is supposed to be that the "remaining subject-matter is disclosed in the application", but what does that even mean? The Visser insists (in the page about 123(2)) that when a document discloses group A and subgroup B, it does not automatically "disclose" also A minus B. But how do carry out this test then and how to consider the result unambiguous enough for the pre-exam?

      Delete
    4. My take on this is that this is a "disclosed disclaimer" or "positively disclosed disclaimer" (zinc oxide is disclosed in Par. [010]. Here G2/10 is relevant (i.e. remaining subject matter is directly and unambiguously disclosed in application as filed). Delimiting a claim based on A54(3) prior right is only relevant for "undisclosed disclaimers". I have answered that the answer is true.

      Delete
    5. @Charlotte, indeed use of 'zinc-oxide' is disclosed in the application as filed making it a 'positively disclosed disclaimer', which means that according to G2/10 the question to be asked is wether the remaining claimed subject-matter is explicitly or implicitly, but directly and unambiguously, disclosed in the application as filed? We expect the answer to be true.

      Delete
    6. Can be any of question 14 be allowable under 123(2) if no mention of M as 10% is included in claim II-1?. I have not now the text, but were not such feature desclosed as essential?.

      Delete
    7. Sorry, M between 5% and 10%

      Delete
    8. Hi @Nico,
      Thanks for your inputs with respect to Q14.1. However, I respectfully disagree. I believe the correct answer will be “False”.
      G2/10 made clear that such amendment (i.e. if the remaining claimed subject-matter is explicitly or implicitly, but directly and unambiguously, disclosed in the application as filed) should be examined as any other amendment.
      Thus, if we analyze the subject matter of claim II-3, it would appear that it seeks protection for a glass composition for use as a photochromic lens comprising: silicon dioxide, component A, component X, and component M, wherein component M is not zinc oxide.
      It is my opinion that the application as filed does not disclose explicitly or implicitly, but directly and unambiguously such a glass composition of claim II-3. In other words, the application as filed does not disclose a composition wherein component M can be ANY material except zinc oxide.
      Could you please further elaborate why do you expect the answer to be “True”?
      Thanks for your time! Indeed very much appreciated your answers/comments!

      Delete
    9. Im struggling with the same question. Some guidance can be found by reading G2/10 itself. In r.2.3, r.4.5.3, r.4.5.4 the Board alleges repeatedly that disclaiming only a narrow, specific embodiment "usually" does not poise a problem under 123(2).
      This appears to be seen as a "normal" limitation of the scope (see r.4.6).
      Maybe one could see it similar to deleting an element from a long list. There, it is usually also not explicitly disclosed in the application that said list may have the form "everything minus element X", but 123(2) conformance of that is never questioned.

      Still, Im not fully convinced that the answer is unambiguous enough to ask in the pre-exam. In contrast to the established "rules" for dealing with amendments concerning lists etc., "rules" how to deal with disclosed disclaimers do not appear to exist, e.g. there is nothing helpful to be found in the Guidelines or the Case Law Book.

      Delete
    10. Lukáš Drevený7 March 2021 at 22:24

      Dear all,

      Thank you for your ideas to Q14.1. Let me give you my explanation, why the answer is false.

      The disclaimer of amended claim is a „disclosed“ disclaimer, because in the application as originally filed the now excluded three-components group of lime, zinc oxide and alumina is described in p10 as such and indicated as being particularly advantageous. Moreover, the group M is mentioned even in the claim 7 filed together with the first European patent application.

      For the admissibility of „disclosed“ disclaimers with regard to Art. 123(2) EPC the criteria given in decision G 2/10 are to be applied (see also Guidelines, H-V, 4.2). Accordingly, the introduction of a „disclosed“ disclaimer offends Art. 123(2) EPC, if the subject-matter remaining in the claim cannot be derived by a person skilled in the art directly and unambiguously from the application as originally filed.

      In the present case the technical teaching of the application as originally filed is that the presence of a zinc oxide is preferred.

      By introducing the disclaimer, this technical teaching is reversed in its opposite and therefore provides the person skilled in the art with “new technical information” (contrary to the requirements of the “modified novelty test”, see T 194/84) with respect to the original application, since the presence of a zinc oxide has to be avoided according to the disclaimer. It follows that the subject-matter remaining in the claim is not derivable “directly and unambiguously, using common general knowledge” (“Gold standard”, compare G 2/10 (see 4.3), Guidelines, H-IV,2.2) from the application as originally filed.

      The disclaimer is therefore not allowable. A corresponding objection might however be overcome by positively claiming the two remaining alternatives.

      Delete
    11. GL H-IV, 4.2:

      "This test is the same as that applied when the allowability of a limitation of a claim by a positively defined feature is to be determined (see H‑V, 3.2)."

      H-V, 3.2:

      "If the resulting combination is novel over the application as originally filed (see the test for novelty given in G‑VI, 2), the amended claim does not fulfil the requirements of Art. 123(2)."

      + H-V, 3.3:

      The deletion of alternatives from more than one list is only allowable if this does not result in the creation of new technical information that is not directly and unambiguously derivable from the application as originally filed.

      and example in there for which:

      The limitation to inhalation in claim 1 results from a choice from one list and has a basis in the application as originally filed.


      Which reflects the novelty concept:

      G-VI, 8 (i)
      Single-list selection concept also applies to single-list deletion.

      Delete
    12. "zinc oxide is preferred" is not the same as "zinc oxide is essential/necessary": it could be something different.

      Delete
  10. Regarding question 16.2 Claim IV.3 meets the requirements of Art. 84 EPC, I do not agree as the feature in question is optional and as such does not restrict the scope of the claim. The scope of the claim as such (without the optional feature) is clear.

    ReplyDelete
    Replies
    1. I don't think so. Only because a part of a claim is not limiting, it does not mean that the clarity requirements do not apply to this part.

      Delete
    2. Clarity applies to the claim as a whole.

      If a claim set reads:
      1. A vehicle comprising a plurality of wheels.
      2. A vehicle according to claim 1, the vehicle being a car, preferably a bicycle.
      then claim 2 is unclear, as a bicycle is not a kind of car.

      Delete
  11. Why should it matter if the feature is optional or not?
    Claim 84 says the claims must be clear, not only their narrowest possible scope.
    The feature is there and it is unclear.

    ReplyDelete
    Replies
    1. I totally agree with @DCF. I actually had the same doubt, because the Delta's P-book always cross-out such feature and comment that this feature doesn't count.

      However, I ask myself, if it happens in a claim in real life, would Examiner raise A.84 objection? The answer is apparently yes and he will definitely ask you to either correct or remove such unclear feature, so I chose NOT CLEAR.

      Delete
    2. I donot agree. We never cross it out if it is about clarity!

      Delete
    3. Sorry Roel, my bad for not making it clear. Because in the P-Book these analysis related to novelty, I shouldn't understand that the same way could be applied to clarity.

      Delete
    4. @Anonymous 3 March 2021 at 22:01

      In our P-book, we do NOT indicate that a for is not limiting. On the contrary, we indicate that a "for" is (usually, depends on the claim type and wording) a "suitable for" and that a "suitable for" surely has a limiting effect: see the ice cube mould and mjould for molten steel in the Guidelines!!!

      Delete
  12. I am not super sure..

    According to the guidelines:
    Optional features
    Optional features, i.e. features preceded by expressions such as "preferably", "for example", "such as" or "more particularly" are allowed if they do not introduce ambiguity. In such a case, they are to be regarded as entirely optional.

    These expressions introduce ambiguity and render the scope of the claim unclear if they do not lead to a restriction of the subject-matter of the claim.

    For example, the wording "a method to manufacture an artificial stone, such as a clay brick" does not fulfil the requirements of Art. 84, because a clay brick will never be an artificial stone. Hence it is unclear if either an artificial stone or a clay brick is manufactured by the method of the claim.

    Analogously, the wording "the solution is heated up to between 65 and 85°C, particularly to 90°C" does not fulfil the requirements of Art. 84 because the temperature after the term "particularly" contradicts the range before it.

    ReplyDelete
  13. Agree, preferably statements are allowed if they are entirely optional and do not introduce ambiguity. According to Fig.1 is a particular disclosure of the flat base feature (e.g. rectangular not hexagonal).

    The reference to a figure is ruled out under Rule 43(6) EPC but not by Article 84 (which does not refer to the Rule 43(6) or the implementing regs). So by the wording of the question the answer is FALSE.

    ReplyDelete
    Replies
    1. Sorry, meets the requirements Article 84 EPC but fails Rule 43(6) EPC. So TRUE.

      Delete
    2. Rule 43 implements Article 84.

      So if failing Rule 43, then failing Article 84.

      Delete
    3. Dear Anon
      "The reference to a figure is ruled out under Rule 43(6) EPC but not by Article 84 (which does not refer to the Rule 43(6) or the implementing regs). So by the wording of the question the answer is FALSE"

      I disagree. Regarding the implementation of Rule 43(6), CLBoA ii(a)2.4 discusses both clarity and conciseness which reads directly onto the wording of A.84.

      Delete
    4. Just as a thought without any legal basis. It is specifically said in the description that the base is not shown in Fig.1. I would consider a reference to something that is not there as unclear, irrespective of rule 43(6).

      Delete
    5. The fact that Fig 1 does not show the base would seem to be crucial. Good point!

      Delete
    6. There is no need to show the base - it clearly has a base.

      So, what exactly YOU can see in the Fig is not so much of relevance, it is what object is shown: if you see a figure of a ball, you see a circle, but it shows a ball!

      Delete
    7. Thanks Roel. But what of Anon 2 March 11.08's comment re:the difference between R43(6) and A84? Seems over-pernickety but maybe correct?

      Delete
    8. Hi BaconAndEggs,

      There is no difference between R.43 and Art.84: R.43 rather implements Art.84 by giving various detailed requirements, including R.43(6) (reading: "Except where absolutely necessary, claims shall not rely on references to the description or drawings in specifying the technical features of the invention. In particular, they shall not contain such expressions as "as described in part ... of the description", or "as illustrated in figure ... of the drawings".")

      So, if failing Rule 43(6), the failing Art.84.

      The way the EPO interprets and applies it is described in more detail in GL F-IV, 4.17:

      Part F – The European Patent Application
      -> Chapter IV – Claims (Art. 84 and formal requirements)
      ---> 4. Clarity and interpretation of claims
      -----> 4.17 References to the description or drawings

      "As indicated in Rule 43(6), the claims must not, in respect of the technical features of the invention, rely on references to the description or drawings "except where absolutely necessary". In particular they must not normally rely on such references as "as described in part ... of the description", or "as illustrated in Figure 2 of the drawings".

      The emphatic wording of the excepting clause is to be noted. The onus is upon the applicant to show that it is "absolutely necessary" to rely on reference to the description or drawings in appropriate cases (see T 150/82).

      An example of an allowable exception is an invention involving some peculiar shape, illustrated in the drawings, but which cannot be readily defined either in words or by a simple mathematical formula. Another special case is that in which the invention relates to chemical products some of whose features can be defined only by means of graphs or diagrams."

      Delete
  14. Q14.3 "Claim II-4 is allowable under Article 123(2) EPC": The amended claim uses the wording " content of 10% to 30% by weight." In the original claim I.2 the wording was " content of about 10% to about 30% by weight."

    I do not agree that this is an allowable amendment, because it leaves out the words "about". There is disclosure for this in the original description. There is a basis for a more precise definition, such as content of 10%-20% or for content of 16%, but not for "content of 10% to 30% by weight". If they would like to amend this, they should replace it, if possible, by a more precise wording found elsewhere in the disclosure as originally filed. And if they cannot, they could retain it in the claim.

    GL, F-IV, 4.6.4, last paragraph : "Where the relative term has no well-recognised meaning the division invites the applicant to replace it, if possible, by a more precise wording found elsewhere in the disclosure as originally filed. Where there is no basis in the disclosure for a clear definition and the term is no longer the only distinguishing feature, it may be retained in the claim, because excising it would generally lead to an extension of the subject-matter beyond the content of the application as filed - in contravention of Art. 123(2)."

    ReplyDelete
    Replies
    1. I totally agree with you. I have also followed this approach

      Delete
    2. I was also thinking about it when I saw that the "about" is missing in the amended claim. But I made similar amendments many times in real life and never received an A123(2) objection. So I followed the more practical approach.

      Delete
    3. Well about is not limiting, see https://www.epo.org/law-practice/legal-texts/html/guidelines/e/f_iv_4_7_1.htm (i.e. the expression "about 200°C" is interpreted as having the same round-off as "200°C"...) so there is no difference

      Delete
    4. Speaking from real life experience, when the application was drafted in the US (where they use "about" a lot), the Examiner typically asks you to delete the term about. This would not be such a standard objection/response to the objection if this amendment were against Art 123(2) EPC

      Delete
    5. Our answer is based on the same interpretation as the one given by ms:

      GL F-IV, 4.7.1: "i.e. the expression "about 200°C" is interpreted as having the same round-off as "200°C""

      As there is no errors margins given in the application and no specific meaning of the term "about", the "about" has no meaning.

      Same principles apply as to what is disclosed for amendments as for prior art, so this irrelevance of "about" also applies here.

      Delete
  15. 19.2 - I am not sure I agree with this one. Embodiment 1 was flat and rigid, and was rigid to make is safer (not safe). Something not rigid would be safe to a degree...

    More importantly, Embodiment 2 is flat, but is not rigid as a whole by virtue of the folds/hinges. So there is a good argument that flat is not tied to rigid.

    I think that amendment is therefore arguably allowable even if the spec and questions were really trying to push you towards FALSE...

    ReplyDelete
  16. Can someone please explain to me why Q13.2 is FALSE? I think D1 is novelty destroying as paragraph [007] in D1 discloses a preferred embodiment with 4 components (one being silicon dioxide). Claim I-1 discloses Components A, M and X, and D1 discloses Components P, B and T. However, no further features are disclosed in the claim or D1 to distinguish these three features, so they can be the same = D1 is novelty destroying?

    ReplyDelete
    Replies
    1. Claim I-1 requires the combination of silicon dioxide, A, M and X. D1 includes a table with a variety of examples of components with which sand (primarily silicon dioxide) may be combined. However, to obtain the claimed combination from D1 you would need to select each component from a different list. Selection from multiple lists renders the claim novel - the specific claimed combination is therefore not disclosed in D1.

      Delete
    2. Thank you - I understand this, but my point is that Components A, M and X are not specific compounds - they’re just labels given to a group of possible compounds? I agree with your point completely if the components disclosed were specific compounds, but in the description, for example, Component A can be sodium oxide, sodium carbonate or boron trioxide. I have therefore construed Claim I-1 to disclose: Silicon dioxide and three other components. The description and D1 are not linked, so surely it doesn’t matter what alphabetical labels are given to the different components. Since the components are not specific compounds, surely it is not absolutely necessary that ‘Component A’ in the description is the same as ‘Component A’ in D1? I therefore still think that the components disclosed in the preferred embodiment of D1 (Components P, B and T) can be the ‘same’ as Components A, M and X in Claim I-1!

      Delete
    3. I think M brings a relevant point regarding labelling of components. I was also pondering quite a bit about whether the labelling is arbitrary or not. I made an assumption that most likely the intention of the examination committee was to make it non-arbitrary. I do feel, however, that such things should be clear from the beginning, and a disclaimer clearly stating that "component A", "component M", "component X"... etc. are not arbitrary is necessary in this case. After all we have seen examples in the past (see e. g. PreExam 2019, Part III - Chemistry, which by the way was a very elegant assignment) when "a first layer", "a second layer" and "a third layer" were considered arbitrary, i. e. "first", "second" and "third" did not specify the order.

      Irina

      Delete
    4. In D1, there is a list of stabilisers (M, N or P), a list of additives (B, A or C) and a list of UV light absorbers (T, U or X). Regardless of whether a particular component can be further specified, in order to have a composition comprising M, A and X, the skilled person would still need to make selections from multiple lists provided in D1.

      Delete
    5. This is all true and within the framework of the exam, all of this should very clear, to be honest. I have made the same mistake as M, though.
      In real life, a claim defining the presence of SiO2 plus a "component A", would, given the absence of any further information about said component, be interpreted so that "component A" as a term is virtually meaningless. Any composition comprising SiO2 and some other random component would fall under said claim. Even D1, because regardless how D1 defines component A or other components, this cannot change the interpretation of the claim itself.

      While I confess this might be more of a brain fart than a valid interpretation, I am still slightly disappointed by the drafting of the exam here. This confusion could have easily been avoided by replacing "component A" with a term like "UV adsorber" or the like, just as it was done in D1.
      Especially given that in previous exams, the candidates where required to critically question whether certain nomenclature like "first layer" or "counter-clockwise" was really limiting or not.

      Delete
    6. @DCF: "component A" is not just any component - it is a certain type of component, for which the character A is used as a reference rather than a terribly complex chemical structure formula. This is done in almost every exam.

      "Component A" as well as other components appear in the various documents - they are the same in each document if referred to by the same name.

      Delete
  17. I am a bit confused with Q3.2: Of course divisional has same date as parent, but is that not ONLY when the subject-matter does not extend beyond the parent? Art.76(1) read :"It may be filed only in respect of subject-matter which does not extend beyond the content of the earlier application as filed; *in so far as this requirement is complied with*, the divisional application shall be deemed to have been filed on the date of filing of the earlier application and shall enjoy any right of priority. " Therefore in this case the Div2 cannot be regarded a divisional, but a new application with new filing date?

    ReplyDelete
    Replies
    1. Hi Anon, check out Visser Art. 76(1):10, 3rd para.

      Delete
    2. To complement Andy's answer: a divisional cannot get its own filing date as A76(1) only defines one date for a div, i.e. the filing date of the parent (G1/05). Claims having SM extending beyond the parent do not get a filing date but have to be amended out of the application (G1/05, GL C-IX, 1.4). Div cannot be converted into an independent app taking its own date, nor can a further div be filed.

      Delete
    3. I see, thank you!

      Delete
  18. Hello everibody,

    Thanks for your work !

    Relating to Q17.3, I believe there is a mistake.

    Claim IV.6 relates to the cover part and not to the receiving part of the protection device. I agree that the receiving part of D1 is a flat base but this is not the object of the claim.

    We do not however have information about the cover part. I believe the statement is false, D1 does not destroy the novelty of claim IV.6.

    Have a nice day,

    ReplyDelete
    Replies
    1. Hi Dieu,

      there are many different ways of visualising what is the cover part and receiving part for D1. One such way is that the cover part is the box and the receiving part is the silicone holder within the box. In this interpretation the cover part has a flat and rigid base, therefore comprises a flat and rigid section. I acutally got this question wrong in the exam as I messed up the claim dependencies due to too much scrolling.

      Hope this helps.

      Delete
    2. I accidently got this one right I guess. I considered the lid of the shipping box to be the cover part and I was struggling because the lid was only described as being rigid but not to comprise a flat section. However, I reasoned that the box being a shipping box implies that the lid is at least partly flat, since the point of a shipping box is to fit as many boxes as possible in a shipping container. Glad to see there was an easier solution to the problem :)

      Delete
    3. Anonymous - the lid not being described as flat was my reasoning for saying false, but thats because I was thinking of a claim that also contained a hinge. If there must be a hinge between the cover and receiving part then the interpretation I gave above isn't possible. Oh well...

      Delete
    4. Hi James,

      I did not think about this interpretation and this is now clear to me.

      Thank you

      Delete
  19. I have a couple of questions about claim analysis answers. I didn’t do terribly but I don’t agree with some of the answers provided on the blog. For instance, the below:

    11.4 – it is not clearly disclosed that silicon oxide is A in the claims, while it is in the description it is not in the claims. The extent of the protection conferred by an EP application shall be determined by the claims (Art. 69 EPC).

    16.2 – comprises a hinge is not limiting but it should state “at least a hinge” or “a first hinge”

    17.3 – 4.6 would be novel it is limited to the claims which it is novel over… in the broad dependency, it is not novel. Equally not novel as it is novel, based on the comments provided by DP.

    17.4 – “a hinge” is not the same as four hinges (see reasoning above)

    18.4 – D2s peripheral wall is not the same as a corner, a plurality of corners and a wall is not the same.

    18.2 –said feature has to be derivable directly and unambiguous from D2, for which it is not.bngbgf

    ReplyDelete
    Replies
    1. Apologies for the typo in my previous post. Does anyone else have similar views on the above questions?

      Delete
    2. Hi Katrina,
      With regard to your comment on 11.4, the statement says whether the claim *covers* a glass composition with the listed components. In other words, would such a glass compoisition fall within the scope of the claim? The claim is indeed broader than this, but this composition would indeed fall within the scope of the claim.

      For 17.3, novelty is assessed based on the broadest dependency - if any dependency lacks novelty, the entire claim lacks novelty. If you have a claim dependent on "any preceding claim" which lacks novelty when depending only on claim 1, but is novel when depending on, say, claim 3 and claim 1, then the claim lacks novelty but can be amended to depend specifically on claim 3.

      For 17.4 - if your claim includes "a hinge", then a further limitation could be having multiple hinges.

      For 18.4 - according to D2, the receiving part is made from a single piece of silicone, which means that the corners are also made of silicone, which is elastically deformable.

      For 18.2 - the hinges being parallel is implicit from D2 and can also be seen in the figure provided in D2.

      Delete
    3. Hi, I agree with Katrina. It was not directly and unambiguous from D2 that the hinges are parallel. I disagree with Stefanie that this can be seen in the fig. of D2. The fig. of D2 is a perspective view where only one side is shown. Hence, one cannot conclude if the hinges are parallel without having a view of the other side. I think the answer should have been True here.

      Delete
    4. D2 discloses that the cover part has three flat and rigid parallel sections joined together by hinges. So the sections are parallel. D2 also discloses that the sections can be folded into a triangular prism shape, as illustrated in Fig. 3. From this it follows that the hinges must be parallel to each other. However, this is slightly ambiguous, but based on this reasoning, we still expect the answer here to be false.

      Delete
    5. I think it is only ever so slightly ambiguous, if at all.
      I find it funny however, that both Figure 2 of the application as well as the Figure of D2 clearly show that the prism is folded from 4 section of the cover, even though both descriptions explicitly say that there are 3 sections. However, this contradiction does not appear to be relevant for any of the questions.

      Delete
    6. D2, [004] As shown in Fig. 3, the cover part (100) is divided into three flat and rigid parallel sections which are joined to each other by hinges These sections can be folded into a triangular prism shape serving as an inclined support for the receiving part holding the tablet computer (20).

      If the (rigid) sections are parallel, and are joined by hinges, and can be folded into a triangular prism shape, the hinges must be parallel to the sections, and thus parallel to each other.

      Or can it be different? if so, please describe how (Katrina, Sofia).

      Delete
    7. @DCF: you are correct: Fig 2 shows 4 parallel sections. As claim IV.7 says that "the cover part is divided into three parallel sections", the figure would not destroy the novelty (it would be different if the claim would read "the cover part comprises three parallel sections" or "at least a part of the cover part is divided into three parallel sections".

      But... at first sight it appears to be without any doubt that the description of D2 specifies in [004]: "the cover part (100) is divided into three flat and rigid parallel sections which are joined to each other by hinges". THIS is novelty destroying.

      We think that that is how the statement was intended.

      BUT:
      [004] of D2 starts with a half-sentence with a reference to the Figure.
      [004] says: "AS SHOWN IN FIG. 3, the cover part (100) is divided into three flat and rigid parallel sections which are joined to each other by hinges. These sections can be folded into a triangular prism shape",
      whereas the figure clearly and without any doubt 4 sections (as the side of the triangle that is in contact with the back of the screen is double.
      So, you could argue that there is clearly an error in D2: the first half sentence refers to 4 sections (via the Figure) and the second to 3...

      GL G-IV, 9 "Errors in prior-art documents" provides:

      "Errors may exist in prior-art documents.

      When a potential error is detected, three situations may arise depending on whether the skilled person, using general knowledge,

      (i) can directly and unambiguously derive from the prior art document that it contains an error and what the only possible correction should be;
      (ii) can directly and unambiguously derive from the prior art document that it contains an error, but is able to identify more than one possible correction; or
      (iii) cannot directly and unambiguously derive from the prior art document that an error has occurred.
      When assessing the relevance of a document to patentability,
      in case (i), the disclosure is considered to contain the correction;
      in case (ii), the disclosure of the passage containing the error is not taken into account;
      in case (iii), the literal disclosure is taken into account as is"

      The error in [004] is not of type iii), but of type (i) or (ii).
      As both a 3-sided, 3-sections sided prism (as described in [004]) as well as a 3-sided 4-sections side prism (as shown in Fig.3) are possible and achieve the required function, you can argue that we are in type (ii)...

      ...so that when assessing the relevance D2 to patentability, the disclosure of the passage containing the error is not taken into account ([004] and Fig.3]

      You could argue that the Figure as such is clear, and that the error is in the text as that is non consistent with the figure. Then you are in (i): the Figure is the disclosure (3-sided, 4-sections), and [004] first sentence is incorrect and is ignored.

      I (now) tend to the last interpretation. Such that:
      D2, showing a cover part that is divided into FOUR parallel section, is NOT novelty destroying for the claimed cover part that is divided into three parallel sections.

      What are your views?

      Delete
    8. roel

      i expect that it was not noticed that D2 had a discrepancy between the drawings and the description - i would consider the above analysis to be outside the remit of pre-eqe exam.

      what do you think?

      Delete
    9. in fact i think it is just an error by the draughtsman for both fig 2 and fig 3?!

      Delete
    10. Tbh, I would have thought that the discrepancy between Figure 3 and [004] has no consequences at all, because [004] is clearly novelty destroying without having to refer to the drawing at all.
      However, based on what the GL say, I also think that case (ii) would apply. There is clearly an error and there is no telling whether the text or the drawing is wrong.
      If the (rather harsh) consequence of that is to disregard the passage in question, this would mean that 004 and Figure 3 must be disregarded and D2 cannot be novelty-destroying anymore.

      Delete
    11. I actually understood from the text of the invention and of D2 that the section on the back of the receiving part is the hinge connecting the cover part and the receiving part, when these are from two different materials:
      (004) The cover part is joined to a side of the receiving
      part by a hinge which is attached to both the base of the receiving part and to the cover part.
      (001) of D2: The cover part is joined to the receiving part
      by a hinge so that the cover part protects the screen of the tablet computer.
      If the section on the back is the hinge, then the cover part is only the part actually covering the screen and has 3 sections.
      Then figures 2 and 3 represent the case where cover p. and receiving p. are from different materials and are correct.
      When cover part and receiving p. are of the same material, the hinge is only a fold, and the cover p. has without doubt 3 sections.

      Delete
    12. "i would consider the above analysis to be outside the remit of pre-eqe exam"

      Maybe it was not intended, probably not, as there is usually more direct hints as to what to consider. I.e., if it was expected to base your answer on such an analysis, I would expect a phrase like "when considering [004] and Fig. 3 in detail", or even "when considering the discrepancy between the wording of [004] and what is shown in Fig.3" or something similar.

      But it is not wrong and is well-documented in the GL, so if a candidates based his answer on this reasoning, it is a valid reasoning and answers based on this analysis are (also correct, shoudn't they be?

      Delete
  20. Would you consider it likely that the epo considers one out of 15.3 and 15.4 to be true and the other one to be false?

    ReplyDelete
  21. Regarding Q15.2, were we supposed to check Art. 54(3) regarding inventive step argumentation for the disclosed prior art in the Claims part? I don't think that is common practice for the pre-exam and hence I thought that was not expected from us to answer the question with respect to that.

    ReplyDelete
    Replies
    1. @Peny, the dates on the documents are quite clear and hard to miss. It is also quite fair to test A.54(3) in such an "real" applicatoin scenario, at least it is better than asking you in the legal part, giving you several dates, and asking whether it can be used for inventive step or not...

      Delete
    2. @Peny: they give the dates for a reason. So, if you do your prior art analysis, and you have the dates, you as a candidate need to check whether it is a 54(3) or 54(2)

      NB; 54(3)s have indeed not been in the claims analysis part of the pre-exam before, but that was more of a surprise than that they now are!

      Delete
  22. Thank you for the Analysis!

    Regarding
    "17.2 D1 destroys the novelty of the subject-matter of claim IV.2

    True: The bottom part of the shipping box can be considered as a receiving part (“flat base and four walls for receiving the tablet computer”) and the flaps as the cover part; the flaps are attached to the walls via hinges."

    At first I came to the same conclusion.

    But isn't a hinge more specific than a foldable flap? Thus the feature of D1 would be more general and not novelty-destroying. In my opinion a hinge provides at least one degree of freedom to rotate the parts around an axis. Thats not necessary for folding as such because it can also be achieved by plastic deformation or by breaking some areas.

    Also reasonable? What do you think?

    ReplyDelete
    Replies
    1. Well, in the description of the invention the hinges can be formed by folds. So according to the invention, folds are hinges. Therefore, the fold of the foldable flap in D1 can be read as a hinge in the sense of the invention.

      This at least is the reasoning I followed.

      Delete
    2. Thank you. That was my first idea too. I probably changed my opinion because of the German terms "Falz" and "umgeknickt" since they are not so similar, but that wasn't intended as the correct answer:/

      Delete
    3. Based on the EN wording I also first considered whether or not D1 comprises hinges between the flaps and walls. Similar to Anonymous above, I then studied the DE wording which describes the application "hinges" as "Falz/Scharnier" and that the flaps of D1 are "umgeknickt", which features are certainly dissimilar from another, rendering the claim novel over D1. Therefore, I concluded the answer should be "F".

      Delete
    4. Is "umknicken" nor the same as "scharnieren"

      If the flaps and walls are not connected by what is considered a hinge (the fold line), what then?

      See also uwe's comments above, to which I fully agree.

      Delete
    5. I think that a hinge is different from a fold.
      A hinge is per definition a joint or mechanism connecting linked objects, and has a more complex structure than a fold.

      Moreover I think that is improper to interpret a prior art document (D1) on the basis of the content of an invention having a later filing date, or?

      If in D1 the flaps are connected to the walls through hinges, they should have written "hinges" also there.

      Delete
    6. Hi Lisa,

      In real life different applicants commonly use different terms for certain features, so having the same wording doesn't make sense as a test for novelty. So long as the feature described falls under the scope of the claims, then the claim will not be novel.

      The question here isn't whether D1 considers a flap to be a hinge or not, but rather whether the claims of the present invention consider a flap to be a hinge.

      So long as this is the case, which it appears to be ("hinges can be formed by folds"), then folds fall under the scope of the claim, and thus the disclosure of folds in D1 is disclosure of the technical feature "hinges" in claim IV.2

      This discussion is related to the concept of each patent being its own dictionary. I have had opposition cases where the opponent tries to use the definitions as given in the prior art documents instead of the definitions as given in the opposed patent and these arguments are never successful, so long as there is a clear definition in the opposed patent

      Delete
    7. Whoops. All mention of "flap" in my previous post should be "fold"

      Delete
    8. I'd like to specify "later filing date" or "priority date" whatever applies as effective date (in this case not specified.

      To notice, D2 specifies in paragraph [001]: "The cover part is joined to the receiving part by a hinge...", and is therefore unambiguous for this aspect.

      Delete
    9. Thank you James for the very clear explanation.
      However, that explanation cannot apply to the German version where different words/verbs are used, other?

      Delete
    10. Hi Lisa, for the German version it depends on whether the PSITA (having a mind willing to understand) would consider flaps that are "umgeknickt" as being "Falze" (i.e. does it have the same technical meaning - see Roel's point at 10:27). My understanding is that this would be true, but, although I live in Germany, my German isn't perfect - so don't take my word for it :)

      Delete
    11. Hi, I think the PSITA would consider flaps that are "umgeknickt" not as being a "Scharnier durch einen Falz gebildet" because a "Scharnier" can only be "umgeknickt" by breaking it.

      In other words: The term "Scharnier durch einen Falz gebildet" is rather a limitation than a definition since not every "Falz" is suitable as a "Scharnier".

      Delete
    12. The German word "umknicken" is described by www.duden.de as "so umbiegen, dass ein Knick entsteht", which translates as "bend such that a kink/edge occurs". Arguably D1 does therefore not teach that the kink/edge is formed at a predetermined location and would not suffice the definition of a hinge according to the application. A second point that I would like to bring up is, that, in my opinion, D1 does not necessary teach that the hinge is located between the receiving part and the cover part (at least according to the interpretation of the German version of D1), because the edge/kink could be located anywhere inside the flap leaving no part that is entirely outside the edge/kink.

      Delete
    13. I agree with anonymous 5 March 2021 at 19:40. I replied FALSE as the specification in the description that the hinge might be formed by a fold doesn't mean that any fold in prior art anticipates a hinge according to the invention.

      Delete
    14. Hi Maria,

      I do not agree. The matter is not "anticipates a hinge according to the invention", the matter is whether it anticipates the claim.
      If the claim just specifies "hinge", then it can be anticipated by anything in the prior art that can be considered a hinge.
      As you said "the specification in the description that the hinge might be formed by a fold", the fold can be considered a"hinge", so it anticipates.
      If "hinge" has a special meaning in the invention, then that meaning should be specified in the claim if you want to rely on it for novelty.

      Delete
    15. Hi Roel,
      Thank you so much for your reply and for giving us the opportunity to discuss via the forum.
      I do agree that the description says that a hinge might be formed by a fold. However It doesn't mean that a fold is thus a hinge.
      According to the description, the hinge might be formed by a fold only if the 3 parts namely the hinge, the cover part and the base of the receiving are made from the
      same material.
      D1 does not disclose that the flaps and the base are of the same material. Moreover D1 indicates that the box can be made of different materials. Thus the cover part and the receiving part might be formed of different material. Thus saying that according the invention "folds" are "hinges" is wrong in my point of view, any fold should not be considered as a hinge and the description it self imposes at least that the parts have to be formed by the same material. This, at least, appears as essential.

      Delete
    16. Does the claim specify that the hinge, the cover part and the base of the receiving are made from the same material?

      Delete
    17. Regarding the comment above, stating that "the PSITA would consider flaps that are 'umgeknickt' not as being a 'Scharnier durch einen Falz gebildet' because a 'Scharnier' can only be 'umgeknickt' by breaking it."

      As a native German speaker, I disagree with this. "Umknicken" in German does not necessarily imply breaking. For example, there is a type of vehicle steering called “Knicklenkung”, which is used in some types of construction vehicles.

      In this type of steering the vehicle has two parts, which are connected by a hinge. The vehicle is then steered by pivoting the two parts with respect to each other. In this sense, “Knicken” means that the two parts are pivoted with respect to each other, forming a pointed angle (the two parts no longer align in a straight line while being “umgeknickt”).

      My point here is that in a “Knicklenkung” the hinge is not formed by a fold, but it can still be “umgeknickt”. And there is also no breaking.

      The comment above implies that something that is “umgeknickt” cannot be a “Scharnier” (hinge). But actually, the contrary is true. If something cannot be “umgeknickt” there is most likely not a hinge involved.

      The word “knicken” CAN be interpreted as bending something to the point of (at least partial) breaking, as the comment implies. In this case it would be an irreversible act.

      But this is NOT the broadest interpretation of the term. In fact, the German dictionary of record “Der Duden” clearly states that “falten” (EN: folding) is a synonym for “knicken”.

      In everyday language the word “umknicken” may not be used that often in relation to hinges. For example, no one would say that a door is opened by “umknicken”. But in technical fields “umknicken” can be used in relation to hinges, as illustrated by the “Knicklenkung” example.

      In conclusion, a piece of cardboard which is “umgeknickt” forms a fold, which can function as a type of hinge. It may not be the most durable type of hinge, but this is not the point. For me this clearly falls under the scope of the claim.

      Also, I am under the illusion that the people designing the exam always have some kind of “lesson for the future” in mind, when they formulate the questions. In this case the “lesson” is for formulating claims.

      It may be obvious that the tablet enclosure envisioned by the invention is not the same as the simple cardboard box disclosed by D1. But according to the wording of the claim it actually is the same. So, there is a lesson here to think about how limiting the words used in a claim actually are, which is important for our profession.

      I would bet that most of us would not have thought of a cardboard box after reading the claim in view of the description of the invention. And for me this is the whole point of this particular exercise. It is to learn to “think out of the box” (pun intended).

      Delete
  23. Hi, thanks very much for the answers. I have a quick (additional) question about 15.4 which I would argue should be FALSE.

    Apart from the arguments that we have already seen in this thread, I would also argue that the technical effect stated in 15.4 is not actually achieved by the referenced claim. Using compound alpha as compound X "provides a photochromic lens that is the most resistant to colour changes as a result of a change in temperature" (paragraph [012]). This paragraph does not disclose that that the lens is resistant to temperature. In other words, a colour change still occurs, just less than with the other compounds.

    To state that the lens is resistant to temperature change would be to suggest that no colour change occurs, which is not apparent from the description. As a result, the technical problem is too narrow since the claim does not actually achieve the technical effect underlying the technical problem.

    Hope this explanation makes sense. Looking forward to hearing what you think!

    Regards,
    Mr Y

    ReplyDelete
    Replies
    1. Hi Mr Y,

      I disagree with this interpretation of "resist" - being resistant doesn't mean that the colour change is completely stopped, only that it is limited in some way. Taking a real life example, friction resists the motion of two objects against each other, but the objects can still continue move if they overcome the resistive force i.e. the motion is being resisted, but can still continue despite that.

      (I don't know if that actually helps with the interpretation of "resist" but it makes sense to me at least!)

      Delete
  24. Question 11.1. On the basis of the claim formulation according to A. 54(5) (Substance X for use...), I thought that "for use as a photochromic lens" is limiting. This formulation is different from "for photochromic lens", which is not limiting (and was used in the past pre-EQE). So I put TRUE.
    Anyone agrees or can comment on this?

    ReplyDelete
    Replies
    1. No, I don't agree. For should be understood as "suitable for" and not limiting, which is explained in Delta's P-Book in many years past exams. Maybe you are thinking about the "use claims"?

      Delete
    2. my understanding is that subsance for use as you've described is for first and second medical use so doesn't apply here.

      Delete
    3. @Anonymous 3 March 2021 at 22:01

      In our P-book, we do NOT indicate that a for is not limiting. On the contrary, we indicate that a "for" is (usually, depends on the claim type and wording) a "suitable for" and that a "suitable for" surely has a limiting effect: see the ice cube mould and mjould for molten steel in the Guidelines!!!

      Delete
    4. Art.54(4) and (5) formulations, novelty can only be achieved if the substance or composition is claimed for use in a method that is excluded under Art.53(3).

      "for use as a lens" is NOT "for use in a medical method" - so, it is not an Art.54(4) or (5) second medical use claim

      Delete
  25. Regarding question 19.4
    does anyone else think that the absence of "and four elasticated corner" in the statement "the receiving part comprising a base, the base being flat" constitute and intermediate generalisation?

    ReplyDelete
    Replies
    1. No, there is basis in the dependent claims.

      Delete
  26. Regarding question 12.2, is the claim unclear because of its dependency or because of its claim category?
    Thanks

    ReplyDelete
    Replies
    1. If it would not be a dependent claim, would the category then be clear? (yes)

      What is the effect of the dependency? Does it render the category unclear? (yes)

      So, it is due to the dependency.

      Delete
  27. We have compiled a version of the full paper (in English); it is available via the link that I added to the blog post.

    In our version, the sequence of the questions and statements corresponds to the order used in this blog, as some of the candidates got them in the real exam; other candidates obtained the questions (in the legal parts) and the statements within a question (in the legal parts as well as in the claims analysis parts) in different order.

    ReplyDelete
  28. Goodmorning,

    First of all, thank you for providing your opinion on the pre-exam's questions. Very much appreciated and very useful.

    Then, there is some disagreement about the answer to question 12.2:
    12.2 Claim I-7 is unclear due to its dependency on claims I-1 to I-6

    The solution seems to be false as it is believed that claim 7 is indeed unclear but for another reason. Claim 7 is formulated as a process, but if we read the whole claim, it seems that the features therein are the natural continuation of claims 1-6, which are directed toward an apparatus, rendering it a problem of category instead of dependency. Claim 7 per se does not mean anything, but if we replace the process word with the apparatus of claims 1-6, claim 7 seems to be clear.
    The conclusion then is that claim 7 is unclear because of its category, not because of its dependency.

    I hope you can come back to me with your comments and your opinion.

    Best regards,
    RC

    ReplyDelete
    Replies
    1. Our answer says:

      True: Claims I-1 to I-6 are composition (product claims) and do not relate to a process; a claim stating “the process according to any one of claims I-1 to I-6” is thus unclear.

      So, I-7 is unclear because the PROCESS claim is phrased to be dependent on a PRODUCT claim; so, the dependency causes the unclarity as to the category of the claim.

      Does that answer your question? If not, please clarify your question.

      Delete
    2. I came to the same conclusion, but did not see that it was different claim categories.

      I thought that since claim 5 was unclear and claim 7 was dependent on claim 5 it must also be unclear. @Roel, if we were to disregard the claim categories would such an argumentation hold

      Delete
    3. Thanks for your prompt reply.
      Yes, indeed I understand your point of view.

      However, if I still don't agree, because:
      if you had changed the dependency of claim 7 into "dependent on claim 9" (which was a process) for example, would then claim 7 be clear? I don't think so, because the features of claim 7 are still directed towards the product of the preceding claims, i.e. a detail of component M, not to the process.
      You still don't agree?

      Regards, RC

      Delete
    4. @Roel, I had a similar reasoning to RC and I appreciate your input in this matter.
      However, instead of constructing the question the way you did, I asked the following questions:

      If we replace the category of the claim (process to product) while keeping the dependency, would the claim be clear and the answer was YES.

      On the other hand, if I change the dependency (instead of "according to claim 1-6", I replace with "according to claim 8" which is directed to a process), the claim will still be unclear, because claim 7 cannot be dependent on a claim that has yet to be announced.

      In view of the above, I concluded that the lack of clarity was not due to the dependency and answered FALSE.

      I would love to have your opinion on this.

      thanks for your terrific job!

      CM

      Delete
    5. I totally agree with CM, as I wrote above in my second comment :)

      Delete
    6. I said true for the same reason as pointed out by ms. If the dependency of claim 5 is incorrect the dependency of claim 7 must be incorrect, too.

      I noticed the change in category when I first read the claims and then overlooked it when I was answering the question :/ luckily this doesn't seem to have any consequences.

      Delete
    7. Hi, I think what is being missed here is that the claim category forms part of the dependency. As in, for the claim to be clear the dependency of the claim requires two parts to be correct - the claim number (and antecedent basis) and the category of the claims. So by one of these parts being incorrect, in this case the category, the whole dependency of the claim is incorrect and the claim is therefore unclear. And so the answer is TRUE - I-7 is unclear because the dependency is incorrect (by virtue of the category part of the dependency being wrong).

      Delete
    8. Thanks for your input @RC.
      The confusion arises from the fact in real-life practice, they are several instances in which an objection under clarity arises from a wrong or confusing claim category (unrelated to the dependency), albeit not always similar to this specific context.

      Furthermore, the GL F-IV-3.4 states that:

      "Also, in some cases, a dependent claim may define a particular feature or features which may appropriately be added to more than one previous claim (independent or dependent). It follows that there are several possibilities: a dependent claim may refer back to one or more independent claims, to one or more dependent claims, or to both independent and dependent claims.@

      Therefore, the category of a claim cannot always be linked to its dependency.
      In fact, it is not uncommon in the biotech/chemistry field to find claims in a given category (for example, a device/apparatus/ claim) referring to other claims in different categories (for example, for carrying out the methods of claims a,b,c, using the product of claims d,e,f).

      In my opinion, the question to ask is: if fixed, would it make the claim clear?

      If we change the category, would it make the claim clear?
      If we change the dependency, would it make the claim clear?

      Thank you all for your comment.
      CM

      Delete
    9. You need to answer the statement as it it with the claims as they are.

      So you cannot disregard the claim categories when you check dependency beung correct or not.

      The problem is in the Claim I-7 wording
      "The process according to any one of claims I-1 to I-6, ...",
      i.e.
      "The process according to any one of the compositions of claims I-1 to I-6, ..."
      i.e.,
      "The process according to the composition, ..."

      That clearly is absolutely unclear, isn't it?

      Delete
    10. Is the claim unclear because of an incorrect choice of category? yes

      Is the claim unclear because of an incorrect dependency? also yes

      The claim is unclear due to the combination of the claim category and the dependency, therefore you can't say that it isn't because of the dependency - without the dependency there wouldn't be a problem. Even if you consider the inclarity as a combination of the two things, the answer to the exam question still has to be TRUE, because the dependency causes the inclarity.

      Delete
  29. Just to detail a bit more, if the GL says that a dependent claim can refer back to multiple independent and dependent claims (knowing that you cannot have more than 1 independent claim per category, it means that a dependent claim can be dependent on claims in different categories. Accordingly, one cannot always link a fault in claim category to its dependency.
    CM

    ReplyDelete
    Replies
    1. A claim that refers to a claim in a different category is NOT a (proper) dependent claim!

      See Guidelines F-IV, 3.8:

      A claim containing a reference to another claim is not necessarily a dependent claim as defined in Rule 43(4). One example of this is a claim referring to a claim of a different category (e.g. "Apparatus for carrying out the process of claim 1 ...", or "Process for the manufacture of the product of claim 1 ..."). Similarly, in a situation like the plug and socket example of F‑IV, 3.2(i), a claim to the one part referring to the other co-operating part (e.g. "plug for co-operation with the socket of claim 1 ...") is not a dependent claim.

      Delete
  30. Reminder:

    Please do not post your comments anonymously - it is allowed, but it makes responding more difficult and rather clumsy ("Dear Mr/Mrs/Ms Anonymous of 05-03-2021 16:56"), whereas using your real name or a pseudonym is more personal, more interesting and makes a more attractive conversation.

    You do not need to log in or make an account - it is OK to just put your (nick) name at the end of your post.

    Thanks!

    ReplyDelete
  31. When do we get our marks?

    When do we get the Examiner’s Report? Same day, earlier or later?

    Which statements will accept both True and False?

    ReplyDelete
    Replies
    1. anywhere between 2 (weeks shortest time) to around a month. I think that this year may be in the longer range, since they would have to consider individual excumstances and the secretary has been overwhelmed with e-mails. But I could be wrong

      Delete
    2. In 2018 and 2019, Pre-Exam results became available 3-4 weeks after the exam:
      - 2019: Pre-Exam on 25/2/2019 - results on 12/3/2019; Examiner's report on same day;
      - 2018: Pre-Exam on 26/2/2018 - results on 21/3/2018; Examiner's report on same or next day (and updated in June due to -presumably interlocutory revision of- appeals).

      A bit of a delay may be in your advantage: then the Committee and Examination Board have more time to take any input into account to consider, for example, neutralization of some statements.
      So although the marking can in principle be quick now since the answers are electronically (in earlier exams, the answer sheet with the colored circles had to be collected, scanned, checked - partially by hand, ...), the careful marking and evaluation process needs some time.
      I agree with ms that this year there may be more time needed in view of the individual circumstances when doing the exam online and the (probably large) amount of emails to the EQE secretariat for Pre-Exam and Main Exam papers (which are all forwarded to the Examination Board -see "Notice from the Examination Secretariat concerning the conduct of the EQE 2021" of 10 March 2021-, who needs to decide on neutralization and on each of the pass/fail decisions.

      Our post and the comments give some indications as to which statements would be candidates for neutralization. In the claims analysis part, I think 15.3, 15.4, and 18.2 (in our version) are candidates for neutralization - I refer to above for arguments for TRUE as well as for FALSE for those statements.

      Delete
  32. Results are available via MyEQE!

    ReplyDelete
  33. Some candidates informed me that the individual Pre-Exam results are available via MyEQE now).

    The Examiner's Report is not yet available, nor is a full list of marks which would allow to determine the pass rate.

    ReplyDelete
  34. The Examiner's report is now available.

    In the claims analysis part (in our numbering), only statement 12.3 (ExRep 12.4) “low”) was neutralized.

    Our answers to all statements in Q.11-Q.20 correspond to that of the Examiner's report.

    Statements 15.3 and 15.4 (Ex.Rep 15.1 and 15.3) were not neutralized; our answers correspond to the official answer. The Examiner’s Report does not comment on a possible difference between “(a glass composition for use as) a photochromic lens” and “a glass composition suitable for making a photochromic lens”, i.e., to the difference between a product/device (lens) and a composition/chemical stuff, - see the discussion on the blog on how that changes the answer.

    Statement 18.2 (Ex.Rep 18.1) as not neutralized. The examiner’s report does not explicitly address the “parallel” aspect nor that the figure shows a triangle made of 4 sections rather than 3. The Examiner’s report just provides “In D2, the cover part is described as divided into three foldable parts forming a triangular prism, as clearly shown in Fig. 3 of D2. Thus, the subject- matter of claim IV.7 is not novel over D2.”

    Note that in Q.11, our version had the somewhat strange order "claim I-1 – I-8 – I-9 (ref I.8) – I-4" whereas the Examiner's report had the natural order "I-1 – I-4 – I-8 – I-9 (ref I.8)" - which probably has less risk of going wrong. Some candidates may have had an even more awkward order, e.g. “I-9 (ref I-8) – I-4 – I-1 – I-8” which seems significantly more easily prone to errors.

    ReplyDelete
1 – 200 of 210 comments Newer Newest